Find the modulus and argument of ##\dfrac{z_1}{z_2}##

  • Thread starter Thread starter chwala
  • Start date Start date
  • Tags Tags
    Argument Modulus
Click For Summary
SUMMARY

The discussion focuses on finding the modulus and argument of the complex fraction \(\dfrac{z_1}{z_2}\), where \(z_1 = 2(\cos\dfrac{π}{3}+i \sin \dfrac{π}{3})\) and \(z_2 = 3(\cos\dfrac{π}{6}+i \sin \dfrac{π}{6})\). The solution involves multiplying by the conjugate of the denominator and applying trigonometric identities. The final results are a modulus of \(\dfrac{2}{3}\) and an argument of \(\dfrac{π}{2}\).

PREREQUISITES
  • Understanding of complex numbers and their representation in polar form.
  • Knowledge of trigonometric identities, particularly those involving sine and cosine.
  • Familiarity with the concept of the conjugate of complex numbers.
  • Basic proficiency in manipulating exponential forms of complex numbers using Euler's formula.
NEXT STEPS
  • Study the properties of complex numbers in polar form.
  • Learn about the application of trigonometric identities in complex number calculations.
  • Explore the use of Euler's formula \(e^{i\theta} = \cos \theta + i \sin \theta\) in complex analysis.
  • Investigate the geometric interpretation of modulus and argument in the complex plane.
USEFUL FOR

Mathematicians, physics students, and anyone studying complex analysis or working with complex numbers in engineering and applied sciences.

chwala
Gold Member
Messages
2,827
Reaction score
415
Homework Statement
See attached
Relevant Equations
Complex numbers
π
1682351380704.png


My take; i multiplied by the conjugate of the denominator...

$$\dfrac{z_1}{z_2}=\dfrac{2(\cos\dfrac{π}{3}+i \sin \dfrac{π}{3})}{3(\cos\dfrac{π}{6}+i \sin \dfrac{π}{6})}⋅\dfrac{3(\cos\dfrac{π}{6}-i \sin \dfrac{π}{6})}{3(\cos\dfrac{π}{6}-i \sin \dfrac{π}{6})}=\dfrac{2(\cos\dfrac{π}{3}+i \sin \dfrac{π}{3})}{3}⋅\dfrac{3(\cos\dfrac{π}{6}-i \sin \dfrac{π}{6})}{3}$$

...This will also realise the required result;though with some work by making use of,

##\cos a⋅\cos b-i\cos a⋅\sin b + i\sin a⋅cos b + \sin a⋅\sin b##

##=\cos a⋅\cos b+\sin a⋅\sin b-i\cos a⋅\sin b+i\sin a⋅\cos b##

##=\cos(a-b)-i\sin (a-b)##

for our case, and considering the argument part of the working we shall have,

##=\cos\left[\dfrac{π}{3}- - \dfrac{π}{6}\right]-i(\sin \left[\dfrac{π}{3}- - \dfrac{π}{6}\right]= \cos\left[\dfrac{π}{3}+\dfrac{π}{6}\right]-i(\sin \left[\dfrac{π}{3}+\dfrac{π}{6}\right]##

##=\cos\left[\dfrac{π}{2}\right]-i\sin \left[\dfrac{π}{2}\right]##
 
Last edited:
Physics news on Phys.org
Use \cos \theta + i\sin \theta = e^{i\theta}.
 
pasmith said:
Use \cos \theta + i\sin \theta = e^{i\theta}.
Fine, let me check on this again...
 
##\dfrac{z_1}{z_2}=\dfrac{2}{3}e^{i\left[\dfrac{π}{3}+\dfrac{π}{6}\right]}=\dfrac{2}{3}e^{i\left[\dfrac{π}{2}\right]}##

Therefore

Modulus =##\dfrac{2}{3}##

and

Argument= ##\dfrac{π}{2}##
 
Last edited:
Question: A clock's minute hand has length 4 and its hour hand has length 3. What is the distance between the tips at the moment when it is increasing most rapidly?(Putnam Exam Question) Answer: Making assumption that both the hands moves at constant angular velocities, the answer is ## \sqrt{7} .## But don't you think this assumption is somewhat doubtful and wrong?

Similar threads

  • · Replies 17 ·
Replies
17
Views
2K
  • · Replies 6 ·
Replies
6
Views
1K
Replies
2
Views
1K
Replies
5
Views
2K
  • · Replies 4 ·
Replies
4
Views
1K
  • · Replies 2 ·
Replies
2
Views
1K
  • · Replies 8 ·
Replies
8
Views
2K
Replies
2
Views
1K
Replies
12
Views
6K
  • · Replies 11 ·
Replies
11
Views
2K